Domanda:
Perché le teorie dei campi quantistici relativistici sono molto più restrittive di quelle non relativistiche?
knzhou
2018-03-31 16:24:31 UTC
view on stackexchange narkive permalink

Parte del motivo per cui la QFT relativistica è così difficile da imparare è che ci sono pile di "teoremi vietati" che escludono semplici esempi fisici e intuizione fisica. Una risposta molto comune alla domanda "perché non possiamo fare X in modo più semplice, o pensarci in questo modo" è "a causa di questo teorema di divieto".

Per fornire alcuni esempi, abbiamo:

Ovviamente tutti questi teoremi hanno presupposti aggiuntivi che tralascio per brevità, ma il punto è che l'invarianza di Lorentz è un presupposto cruciale per tutti.

D'altra parte, la QFT non relativistica, come praticata nella fisica della materia condensata, non ha quasi altrettante restrizioni, risultando in esempi molto più belli. Ma l'unica differenza sembra essere che lavorano con un gruppo di simmetria rotazionale di $ SO (d) $ mentre i fisici delle particelle usano il gruppo di Lorentz $ SO (d-1, 1) $, non un grande cambiamento. C'è una ragione intuitiva e fondamentale per cui la QFT relativistica è molto più limitata?

Ho pensato che potesse essere collegato al fatto che il gruppo di Lorentz non è compatto, ma ancora una volta il confronto saliente è in realtà il gruppo galileiano contro il gruppo di Poincaré!
Penso che quasi tutte le cose che hai menzionato siano solo una conseguenza dell'unitarietà, non necessariamente dell'invarianza di Lorentz, anche se forse secondo me l'invarianza di Lorentz sta entrando in qualche modo sottile.Penso che cose come Reeh-Schlieder possano essere affermate in modo non relativistico.
Haag non ha nulla a che fare con l'invarianza relativistica: https://physics.stackexchange.com/q/312389/84967
C'è un teorema di divieto che afferma che non puoi capire intuitivamente perché la QFT relativistica è molto più limitata.
@Dvij ... che è un corollario del più generale teorema di non-go "non puoi capire intuitivamente nulla nella meccanica quantistica" ;-P
@Dvij Riderei se non fosse così vero!
@knzhou Solo un'osservazione: poiché la QFT non relativistica ha restrizioni minori, sospetto che avrà la libertà di consentire implicazioni aggiuntive che sono escluse in una QFT relativistica.Ad esempio, [I bosoni di Goldstone sono necessariamente particelle di spin-0?] (Https://physics.stackexchange.com/questions/305719/are-goldstone-bosons-necessately-spin-0-particles)
Tre risposte:
AccidentalFourierTransform
2018-03-31 22:52:30 UTC
view on stackexchange narkive permalink

Uno dei motivi per cui le teorie relativistiche sono così restrittive è a causa della rigidità del gruppo di simmetria. In effetti, la (parte omogenea) dello stesso è semplice, al contrario di quella dei sistemi non relativistici, che non lo è.

Il gruppo isometrico dello spazio-tempo di Minkowski è \ begin {equation} \ mathrm {Poincar \ acute {e}} = \ mathrm {ISO} (\ mathbb R ^ {1, d-1}) = \ mathrm O (1, d-1) \ ltimes \ mathbb R ^ d \ end {equation} la cui parte omogenea è $ \ mathrm O (1, d-1) $, il cosiddetto Gruppo Lorentz 1 . Questo gruppo è semplice.

D'altra parte, il gruppo di isometria di galileiano spazio + tempo è 2 \ begin {equation} \ text {Bargmann} = \ mathrm {ISO} (\ mathbb R ^ 1 \ times \ mathbb R ^ {d-1}) \ times \ mathrm U (1) = (\ mathrm O (d-1) \ ltimes \ mathbb R ^ {d-1}) \ ltimes (\ mathrm U (1) \ times \ mathbb R ^ 1 \ times \ mathbb R ^ {d-1}) \ end {equation} la cui parte omogenea è $ \ mathrm O (d-1) \ ltimes \ mathbb R ^ {d-1} $, il cosiddetto Gruppo Galilei (omogeneo). Questo gruppo non è semi-semplice (contiene un normale sottogruppo non banale, quello dei boost).

Esiste infatti una classificazione di tutti i gruppi di simmetria cinematica fisicamente ammissibili (a causa di Lévy-Leblond), che praticamente individua Poincaré come l'unico gruppo con le proprietà di cui sopra. Esiste un'unica famiglia di tali gruppi, che contiene due parametri: il raggio AdS $ \ ell $ e la velocità della luce $ c $ (e tutte le contrazioni invarianti di rotazione İnönü-Wigner degli stessi). Fintanto che $ \ ell $ è finito, il gruppo è semplice. Se si porta $ \ ell \ a \ infty $ si ottiene Poincaré che ha un sottogruppo normale non banale, il gruppo delle traduzioni (e se si quoziente da questo gruppo, si ottiene un gruppo semplice, Lorentz). Se prendi anche $ c \ per \ infty $ ottieni Bargmann (o Galilei), che ha anche un normale sottogruppo non banale (e se fai un quoziente da questo gruppo, non ottieni un semplice group; piuttosto, ottieni Galilei, che ha un sottogruppo normale non banale, quello dei boost).

Un'altra ragione è che il postulato della causalità è banale nei sistemi non relativistici (perché esiste una nozione assoluta di tempo), ma impone forti restrizioni ai sistemi relativistici (perché non esiste una nozione assoluta di tempo). Questo postulato è tradotto nella teoria quantistica attraverso l ' assioma di località, $$ [\ phi (x), \ phi (y)] = 0 \ quad \ forall x, y \ quad \ text {s.t.} \ quad (x-y) ^ 2<0 $$ dove $ [\ cdot, \ cdot] $ denota un supercommutatore. In altre parole, due operatori qualsiasi il cui supporto è disconnesso casualmente devono (super) fare il pendolare. Nei sistemi non relativistici questo assioma è vuoto perché tutti gli intervalli dello spaziotempo sono simili al tempo, $ (x-y) ^ 2>0 $, cioè tutti i punti dello spaziotempo sono casualmente connessi. Nei sistemi relativistici, questo assioma è molto forte.

Queste due osservazioni possono essere applicate ai teoremi che citi:

  • Reeh-Schlieder dipende dall'assioma della località, quindi non sorprende che non si applichi più ai sistemi non relativistici.

  • Coleman-Mandula (vedi qui per una prova). Il gruppo di rotazione è compatto e quindi ammette rappresentazioni unitarie a dimensione finita. D'altra parte, il gruppo di Lorentz è non compatto e quindi l'unica rappresentazione unitaria a dimensione finita è quella banale. Notare che questo è utilizzato nel passaggio 4 nella dimostrazione sopra; è qui che la dimostrazione si interrompe.

  • Haag si applica anche a sistemi non relativistici, quindi non è un buon esempio del punto di vista di OP. Vedi questo post su PSE per maggiori dettagli.

  • Weinberg-Witten. Per cominciare, questo teorema riguarda le particelle prive di massa, quindi non è chiaro cosa significano tali particelle nei sistemi non relativistici. Dal punto di vista delle rappresentazioni irriducibili possono essere significative, almeno in linea di principio. Ma non è necessario che corrispondano a rappresentazioni di elicità (proprio perché il piccolo gruppo della quantità di moto di riferimento non è semplice). Pertanto, il teorema si rompe (poiché dipende in modo cruciale dalle rappresentazioni di elicità).

  • Statistiche di rotazione. Come in Reeh-Schlieder, nei sistemi non relativistici l'assioma di località è vuoto, quindi non implica alcuna restrizione per gli operatori.

  • CPT. Idem.

  • Coleman-Gross. Non ho familiarità con questo risultato quindi non posso commentare. Non so nemmeno se sia violato in sistemi non relativistici.


1: Più in generale, il gruppo indefinito ortogonale (o pseudo-ortogonale) $ \ mathrm O (p, q) $ è definito come l'insieme di $ (p + q) $ -matrici dimensionali, con coefficienti reali, che lasciano invariante la metrica con firma $ (p, q) $: $$ \ mathrm O (p, q): = \ {M \ in \ mathrm {M} _ {p + q} (\ mathbb R) \ \ mid \ M \ eta M ^ T \ equiv \ eta \}, \ qquad \ eta: = \ mathrm {diag} (\ overbrace {-1, \ dots, -1} ^ p, \ overbrace {+1, \ dots, + 1} ^ q) $$

Il gruppo ortogonale indefinito speciale $ \ mathrm {SO} (p, q) $ è il sottoinsieme di $ \ mathrm O (p, q) $ con determinante di unità. Se $ pq \ neq0 $, il gruppo $ \ mathrm {SO} (p, q) $ ha due componenti scollegati. In questa risposta, "gruppo di Lorentz" può riferirsi al gruppo ortogonale con firma $ (1, d-1) $; alla sua componente $ \ det (M) \ equiv + 1 $; o al suo sottogruppo ortocronico $ M ^ 0 {} _ 0 \ ge + 1 $. Solo quest'ultimo è semplicemente connesso. La topologia del gruppo è per lo più irrilevante per questa risposta, quindi non faremo distinzione tra le tre diverse nozioni possibili di "gruppo di Lorentz".

2: Si può dimostrare che l'algebra di Galilei disomogenea, a differenza dell'algebra di Poincaré, ha un secondo gruppo di co-omologia non banale.In altre parole, ammette un'estensione centrale non banale.Il gruppo Bargmann è definito precisamente come il gruppo Galilei disomogeneo esteso centralmente.A rigor di termini, tutto ciò che sappiamo è che l'estensione centrale ha l'algebra $ \ mathbb R $;a livello di gruppo, potrebbe portare a un fattore $ \ mathrm U (1) $ come sopra, oppure a un fattore $ \ mathbb R $.Nella meccanica quantistica la prima opzione è più naturale, perché possiamo identificare questa fase con la $ \ mathrm U (1) $ simmetria dell'equazione di Schrödinger (che ha un gruppo di simmetria più grande, il cosiddetto gruppo di Schrödinger).Ancora una volta, i dettagli della topologia del gruppo sono per lo più irrilevanti per questa risposta.

Puoi davvero costruire un controesempio per Reeh-Schlieder?
@RyanThorngren Il concetto di Reeh-Schlieder non relativistico è privo di significato (perché anche il concetto di algebre locali è privo di significato: se $ c \ to \ infty $, tutti i punti sono collegati causalmente).Pertanto, in senso stretto non ci sono controesempi, perché il teorema è incompatibile con la situazione.Ma nel senso di OP, che afferma "... vieta gli operatori di posizione in QFT relativistico", allora il controesempio è semplice: in sistemi non relativistici, l'operatore di posizione $ \ hat X $ è ben definito, Galilei covariante eLocale.Nessun operatore di questo tipo esiste nel regime relativistico.
Vorrei affermare Reeh-Schlieder che $ \ langle O ^ \ dagger (x) O (0) \ rangle \ ge 0 $, con 0 solo per l'operatore di identità.Ciò deriva dalla positività della riflessione.Non so come costruire l'operatore di posizione nella meccanica quantistica a molti corpi.
@RyanThorngren 1) La versione di RS che avevo in mente è quella di [wikipedia] (https://en.wikipedia.org/wiki/Reeh–Schlieder_theorem), cioè che il vuoto è un vettore ciclico (per algebre su insiemi apertiin Minkowski).Non so se questo sia equivalente alla tua versione.2) In linea di principio, prendi semplicemente $ \ hat X = \ bigoplus_i \ hat X_i $, dove $ \ hat X_i = 1 \ otimes \ cdots 1 \ otimes \ hat X \ otimes 1 \ cdots 1 $, dove $ \ hatX $ è nella posizione $ i $.
Penso che siano equivalenti, o almeno la versione su wikipedia segue dall'affermazione che ho fatto.Le osservabili locali possono semplicemente significare quelle il cui supporto nello spazio di Hilbert è limitato nello spazio reale.Non penso che l'operatore X che scrivi sia così limitato, diciamo, a un modello hamiltoniano saltellante.
@RyanThorngren Sicuramente non è limitato.I valori singolari di $ X $ possono assumere qualsiasi valore da $ - \ infty $ a $ + \ infty $, quindi è sicuramente illimitato.Ciò è richiesto da un punto di vista fisico: la posizione di una particella può assumere qualsiasi valore da $ - \ infty $ a $ + \ infty $.
Sean E. Lake
2018-03-31 22:57:11 UTC
view on stackexchange narkive permalink

L'invarianza di Lorentz è anche un contributore indiretto alle restrizioni che la rinormalizzabilità pone alla teoria. La logica dice qualcosa del genere:

  1. L'azione deve essere invariante di Lorentz, quindi il numero di derivate spaziali deve essere uguale al numero di derivate temporali nell'azione.
  2. Vogliamo che l'Hamiltoniano svolga il ruolo di energia (ha un limite inferiore e fornisce stabilità), quindi l'azione non può avere più di due derivate temporali (per ulteriori informazioni, vedere: questa risposta precedente sulla rinormalizzazione ).
  3. Per 1 e 2, il propagatore $ \ Delta (x, y) $ divergerà, necessariamente, come $ \ left ([x ^ \ mu-y ^ \ mu] [x_ \ mu-y_ \ mu] \ right) ^ {- 1} $ come $ x \ rightarrow y $ (equivalentemente, la forma dello spazio della quantità di moto assomiglia a $ (p ^ \ mu p_ \ mu) ^ {- 1} $ per $ p \ rightarrow \ infty $).

È quel terzo passaggio che porta alle divergenze che richiedono la rinormalizzazione e la rinormalizzabilità è molto restrittiva sui termini che l'azione può contenere. Senza l'invarianza di Lorentz, potremmo aggiungere più derivate spaziali senza derivate temporali, produrre un propagatore finito ben comportato e lavorare con una classe di teorie molto più ampia.

Certo, come discusso nella risposta collegata, potresti rilassarti 2, alcuni, ma questo non consente alcuna teoria, solo di più.

Ryan Thorngren
2018-03-31 22:05:59 UTC
view on stackexchange narkive permalink

Ecco una prospettiva (incompleta), principalmente sull'infrarosso:

Per un campo con date cariche sotto Lorentz e tutte le altre simmetrie, esiste essenzialmente solo una teoria con azione quadratica al primo ordine nelle derivate. Per giri interi è $ \ partial_ \ mu \ phi \ partial ^ \ mu \ phi + m ^ 2 \ phi ^ 2 $ e per giri mezzo intero è $ \ bar \ psi \ gamma ^ \ mu D_ \ mu \ psi + m \ bar \ psi \ psi $. Questo è un fatto della teoria delle rappresentazioni, e il fatto che tutto ciò che devi contrarre gli indici dello spaziotempo sono $ g _ {\ mu \ nu} $, $ \ gamma ^ \ mu $, e cose con più indici spaziotemporali. Nota che la forma di queste azioni determina che i semplici propagatori siano quelli relativistici $ 1 / (p ^ 2 - m ^ 2) $ e $ 1 / (p-m) $, rispettivamente.

D'altra parte, se dovessi rompere la simmetria di Lorentz, ad esempio scegliendo un campo vettoriale $ v ^ \ mu $, potresti scrivere termini come $ \ phi v ^ \ mu \ partial_ \ mu \ phi $, che cambierebbe la relazione di dispersione per $ \ phi $ in modo che sia lineare in $ p $ per momenti paralleli a $ v $. Nota che per $ v $ di tipo temporale questo interrompe il gruppo di Lorentz da $ SO (1, d) $ a $ SO (d) $.

Per un campo magnetico applicato $ F_ {ij} $ su fermioni potremmo aggiungere un termine $ \ bar \ psi F_ {ij} \ gamma ^ i \ gamma ^ j \ psi $ che può confondere le statistiche di spin.

Penso che questi nuovi "punti fissi" gaussiani facciano sì che molte cose diventino instabili (nell'IR) quando fai la teoria delle perturbazioni intorno a loro.

D'altra parte, non ci sono molti termini che possono portare a questi termini, e per questo la maggior parte delle teorie che finiamo per studiare nella materia condensata hanno un'emergente invarianza di Lorentz nell'IR. Alcune eccezioni significative sono le teorie con superficie di fermi singolare o altre con miscelazione "UV / IR" che fanno sì che la teoria dei campi veda il reticolo.



Questa domanda e risposta è stata tradotta automaticamente dalla lingua inglese. Il contenuto originale è disponibile su stackexchange, che ringraziamo per la licenza cc by-sa 3.0 con cui è distribuito.
Loading...